Deux intégrales pour ce jeudi 19/01/2023

Fin de partie
Modifié (January 2023) dans Analyse
Comment feriez-vous pour calculer : \begin{align}J=\int_0^\infty \frac{\arctan^2 x\ln^2 x}{1+x^2}dx\quad ?\end{align}
Et, plus simple, selon moi, montrer que : \begin{align}K=\int_0^\infty \frac{\arctan x\ln^2 x}{x(1+x^2)}dx=\frac{\pi^3\ln 2}{8}-\frac{\pi\zeta(3)}{8}\end{align}
NB. Je suis en train de finaliser le calcul de la première intégrale et cela passe, entre autres, par le calcul de la seconde.

Réponses

  • Pour terminer le calcul de $J$ il faut que je calcule aussi \begin{align}\int_0^1 \frac{t\ln^2 t\ln\left(\frac{1-t}{1+t}\right)}{1-t^2}dt\end{align}
    J'ai peur que le résultat soit une expression avec des valeurs de fonctions polygammas. $K$ est nettement plus facile à calculer moins d'une dizaine de lignes de calculs.
  • Je sais montrer que: \begin{align}J=\frac{7\pi\zeta(3)\ln 2}{4}+\frac{\pi}{2}\int_0^1 \frac{t\ln^2 t\ln\left(\frac{1-t}{1+t}\right)}{1-t^2}dt\end{align} J'ai une idée pour calculer la dernière intégrale mais j'ai peur que la méthode échoue. J'avais posté un petit script GP PARI il y a quelques temps pour donner empiriquement la valeur d'une intégrale de degré 4. et le script ne donne pas de valeur pertinente, d'où ma conviction qu'il y a du polygamma là dessous.
  • fjaclot
    Modifié (January 2023)
    Le calcul de J est assez rapide en utilisant le « Feynman trick »
    Sur celui de K je tourne en rond, ne faisant que rendre plus compliqué l’intégrande.
    fjaclot;
  • Fin de partie
    Modifié (January 2023)
    @fjaclot : Tu n'as pas inversé $J$ et $K$ ?
    $K$ peut se calculer , en effet, en introduisant un paramètre.
  • Fin de partie
    Modifié (January 2023)
    En augmentant la précision le script a fini par donner un résultat probable :
    \begin{align}\int_0^1 \frac{t\ln^2 t\ln\left(\frac{1-t}{1+t}\right)}{1-t^2}dt=-\frac{23}{1440}\pi^4-\frac{1}{6}\pi^2\ln^2 2+4\text{Li}_4\left(\frac{1}{2}\right)+\frac{1}{6}\ln^4 2\end{align}
  • Fin de partie
    Modifié (January 2023)
    Le script<br>beta(n)={intnum(x=0,1,(-log(x))^(n-1)/(1+x^2))};lindep4(x)={NOM=["x","Pi^4","Pi*log(2)^3","Pi^2*log(2)^2","Pi^3*log(2)","polylog(4,1/2)","zeta(3)*log(2)","zeta(3)*Pi","log(2)^4","Pi*imag(polylog(3,1+I))","log(2)*imag(polylog(3,1+I))","Catalan^2","Catalan*log(2)^2","Catalan*Pi^2","Catalan*log(2)*Pi","beta(4)","imag(polylog(4,1+I))"];VAL=[x,Pi^4,Pi*log(2)^3,Pi^2*log(2)^2,Pi^3*log(2),polylog(4,1/2),zeta(3)*log(2),zeta(3)*Pi,log(2)^4,Pi*imag(polylog(3,1+I)),log(2)*imag(polylog(3,1+I)),Catalan^2,Catalan*log(2)^2,Catalan*Pi^2,Catalan*log(2)*Pi,beta(4),imag(polylog(4,1+I))];L=lindep(VAL);for(i=2,length(L),if(-L[i]/L[1]>0,print1("+",-L[i]/L[1],NOM[i]));if(-L[i]/L[1]<0,print1(-L[i]/L[1],NOM[i])));}
  • J'ai avancé un peu.
    Je parviens à montrer, calculs faits, que:
    \begin{align}\int_0^1 \frac{\ln\left(\frac{1-t}{1+t}\right)\ln^2 t}{1-t}dt=\frac{\pi^4}{48}-\frac{7\zeta(3)\ln 2}{2}\end{align}
    Il reste à calculer:
    \begin{align}\int_0^1 \frac{\ln\left(\frac{1-t}{1+t}\right)\ln^2 t}{1+t}dt\end{align}
    Mais, en fait, \begin{align}\int_0^1 \frac{\ln\left(1+t\right)\ln^2 t}{1+t}dt\end{align} s'évalue sans trop de difficulté*.
    Donc, il reste à évaluer:
    \begin{align}\int_0^1 \frac{\ln\left(1-t\right)\ln^2 t}{1+t}dt\end{align}

    *: on part de :
    \begin{align}\int_0^1 \frac{\ln^3\left(\frac{t}{1+t}\right)}{1+t}dt\end{align}






  • Fin de partie
    Modifié (January 2023)
    Le calcul de $J$ que j'ai mené (et qu'il me reste à terminer) commence, de façon très surprenante, par une intégration par parties.

    Je suis passé à travers un "mur" que je croyais infranchissable et j'espérais depuis longtemps passer à travers (au moins dans des cas particuliers)
    Ce que je croyais insurmontable est que la dérivée de la fonction $x\rightarrow \arctan^2 x$ n'est pas une fraction rationnelle, la dérivation ne fait pas disparaître le terme $\arctan x$ ce qui est normalement rédhibitoire pour la méthode que j'applique habituellement. Mais ici, on arrive à surmonter les difficultés générées par la présence de $\arctan x$ après l'intégration par parties.

    La même méthode   pourrait peut-être s'appliquer au calcul de \begin{align}\int_0^\infty \frac{\arctan^2 x\ln^4 x}{1+x^2}dx\end{align} Voire, plus hypothétiquement au calcul de \begin{align}\int_0^\infty \frac{\ln^2(1+x)\ln^2 x}{1+x^2}dx\end{align}

  • Fin de partie
    Modifié (January 2023)
    \begin{align}A&=\int_0^1\frac{\ln^2 t\ln(1-t)}{1+t}dt\end{align}
    Quand on va dériver $t\rightarrow \ln(1-t)$ on va obtenir une fraction rationnelle donc, a priori, on peut essayer l'intégration par parties suivante:
    \begin{align}A&\overset{\text{IPP}}=\left[\left(\int_0^t\frac{\ln^2 s}{1+s}ds-\int_0^1\frac{\ln^2 s}{1+s}ds\right)\ln(1-t)\right]_0^1+\int_0^1 \frac{1}{1-t}\left(\underbrace{\int_0^t\frac{\ln^2 s}{1+s}ds}_{w(s)=\frac{s}{t}}-\int_0^1\frac{\ln^2 s}{1+s}ds\right)dt\end{align}
    On a été obligé de prendre pour primitive $\displaystyle x\rightarrow \int_0^t\frac{\ln^2 s}{1+s}ds-\int_0^1\frac{\ln^2 s}{1+s}ds$ pour qu'il n'y ait pas de problème de convergence. On n'a pas besoin, à ce stade, de calculer la dernière intégrale, elle va disparaître dans le cours du calcul.
    \begin{align}A&=\int_0^1 \int_0^1 \left(\frac{t\ln^2(wt)}{(1-t)(1+wt)}-\frac{\ln^2 w}{(1-t)(1+w)}\right)dtdw\\
    &=\int_0^1 \int_0^1 \left(\frac{\ln^2(wt)}{(1-t)(1+w)}-\frac{\ln^2(wt)}{(1+w)(1+tw)}-\frac{\ln^2 w}{(1-t)(1+w)}\right)dtdw\\
    &=-\int_0^1 \int_0^1 \left(\frac{\ln^2(wt)}{(1+w)(1+tw)}\right)dtdw+2\left(\underbrace{\int_0^1\frac{\ln t}{1-t}dt}_{=-\frac{\pi^2}{6}}\right)\left(\underbrace{\int_0^1\frac{\ln w}{1+w}dw}_{=-\frac{\pi^2}{12}}\right)+\\&
    \left(\underbrace{\int_0^1\frac{\ln^2 t}{1-t}dt}_{=2\zeta(3)}\right)\left(\underbrace{\int_0^1\frac{1}{1+w}dw}_{=\ln 2}\right)\\
    &=-\int_0^1 \int_0^1 \left(\frac{\ln^2(wt)}{(1+w)(1+tw)}\right)dtdw+\frac{\pi^4}{36}+2\ln 2\zeta(3)\\
    \end{align}
    Et, on termine en se mettant en situation de pouvoir effectuer une intégration par parties. On commence donc par le changement de variable $z(t)=wt$.
    \begin{align}A&=-\int_0^1 \frac{1}{w(1+w)}\left(\int_0^w \frac{\ln^2 z}{1+z}dz\right)+\frac{\pi^4}{36}+2\ln 2\zeta(3)\\
    &=-\left[\ln\left(\frac{w}{1+w}\right)\left(\int_0^w \frac{\ln^2 z}{1+z}dz\right)\right]_0^1+\int_0^1 \frac{\ln\left(\frac{w}{1+w}\right)\ln^2 w}{1+w}dw+\frac{\pi^4}{36}+2\ln 2\zeta(3)\\
    &=\ln 2\underbrace{\int_0^1 \frac{\ln^2 z}{1+z}dz}_{=\frac{3\zeta(3)}{2}}+\underbrace{\int_0^1 \frac{\ln^3 w}{1+w}dw}_{=-\frac{7\pi^4}{120}}+\frac{\pi^4}{36}+2\ln 2\zeta(3)-\int_0^1 \frac{\ln\left(1+w\right)\ln^2 w}{1+w}dw\\
    &=\boxed{\frac{7\zeta(3)\ln 2}{2}-\frac{11\pi^4}{360}-\int_0^1 \frac{\ln\left(1+w\right)\ln^2 w}{1+w}dw}
    \end{align}



  • fjaclot
    Modifié (January 2023)
    Bonjour FdP,
    Je tourne en rond !
    Quelle IPP fais-tu comme première étape du calcul de J ?
    Par avance merci
    fjaclot
  • Fin de partie
    Modifié (January 2023)
    @Fjaclot.
    Je croyais que ce qu'il fallait prendre comme primitive pour l'intégration par parties était évident  >:)
    \begin{align}J=\int_0^\infty \frac{\arctan^2 x\ln^2 x}{1+x^2}dx&=\left[\left(\int_0^x \frac{\ln^2 t}{1+t^2}dt\right)\arctan^2 x\right]_0^\infty-2\int_0^\infty\left(\int_0^x \frac{\ln^2 t}{1+t^2}dt\right)\frac{\arctan x}{1+x^2}dx\end{align}
    NB. Après, la philosophie du calcul est la même qu'au dessus. Attention à la décomposition en éléments simples, elle introduit des pôles qu'il va falloir neutraliser (c'est-à-dire corriger un peu les différents termes pour qu'on ait des intégrales convergentes)
    PS.
    Tout au long du calcul les bornes $0,\infty$ aplanissent bien des problèmes.
    PS2.
    La présence de l'arctan dans l'intégrale double fait peur à voir (dans le calcul plus haut, on n'avait que des fractions rationnelles hormis un terme en $\ln^p x$) mais les bornes $0,\infty$ vont permettre d'aplanir les difficultés.
  • @Fin de partie   Je pense qu'on peut calculer le K directement sans l'introduction d'un paramètre mais je laisse d'abord etanche vérifier si ce K est déjà référencé dans son moteur de recherche préféré
    Le 😄 Farceur


  • @Gebrane: Sûrement avec des sommes d'Euler mais je serais étonné d'apprendre qu'on puisse calculer cette intégrale ($K$) avec seulement des changements de variable et intégrations par parties dans des intégrales non multiples.
  • gebrane
    Modifié (January 2023)
    @Fin de partie   Bonsoir
    Pourquoi détestes-tu les intégrales doubles, ce ne sont que deux intégrales simples successives.
    J'utilise  $\arctan x=\int_0^1\frac{x}{1+x^2y^2}\ dy\ $, et avoir
    \begin{align*}
    K&=\int_0^\infty \frac{\ln^2x\arctan x}{x(1+x^2)}\ dx=\int_0^\infty \frac{\ln^2x}{x(1+x^2)}\left(\int_0^1\frac{x}{1+x^2y^2}\ dy\right)\ dx\\
    &=\int_0^1\frac{1}{1-y^2}\left(\int_0^\infty\frac{\ln^2x}{1+x^2}\ dx-\int_0^\infty\frac{y^2\ln^2x}{1+x^2y^2}\ dx\right)\ dy\\
    &=\int_0^1\frac{1}{1-y^2}\left(\frac{\pi^3}{8}-\frac{y\pi^3}{8}-\frac{y\pi\ln^2y}{2}\right)\ dy
    \end{align*}  Le reste tu connais.
    Le 😄 Farceur


  • Fin de partie
    Modifié (January 2023)
    @gebrane. Je ne déteste pas les intégrales doubles (j'ai écrit quoi plus haut ?), mais j'essaie d'appliquer un principe de "réponse graduée" en général. Si une intégrale peut être calculée qu'avec des intégrales simples je vais privilégier cette solution à une solution qui fait appel à des intégrales doubles ou des trucs du même genre (séries de Fourier etc.).
    Ton calcul c'est le même auquel je pensais. Cela revient à introduire l'intégrale à paramètre :
    \begin{align}\int_0^\infty \frac{\ln^2x\arctan (ax)}{x(1+x^2)}\ dx.\end{align}
  • Fin de partie
    Modifié (January 2023)
    Un pas de plus :
    \begin{align}B&=\int_0^1 \frac{\ln(1+t)\ln^2 t}{1+t}dt\\3B&=\underbrace{\int_0^1 \frac{\ln^3 t}{1+t}dt}_{=-\frac{7\pi^4}{120}}-\underbrace{\int_0^1 \frac{\ln(1+t)^3}{1+t}dt}_{\frac{\ln^4 2}{4}}+3\underbrace{\int_0^1 \frac{\ln^2(1+t)\ln t}{1+t}dt}_{\text{IPP}}-\underbrace{\int_0^1 \frac{\ln^3\left(\frac{t}{1+t}\right)}{1+t}dt}_{u=\frac{2t}{1+t}}\\
    &=-\frac{7\pi^4}{120}-\frac{\ln^4 2}{4}-\underbrace{\int_0^1\frac{\ln^3(1+t)}{t}dt}_{u=\frac{1}{1+t}}-\int_0^1\frac{\ln^3\left( \frac{u}{2}\right)}{2-u}du\\
    &=-\frac{7\pi^4}{120}-\frac{\ln^4 2}{4}+\int_{\frac{1}{2}}^1\frac{\ln^3 u}{u(1-u)}du-\int_0^1\frac{\ln^3\left( \frac{u}{2}\right)}{2-u}du\\
    &=-\frac{7\pi^4}{120}-\frac{\ln^4 2}{4}+\int_{\frac{1}{2}}^1\frac{\ln^3 u}{u}du+\int_{\frac{1}{2}}^1\frac{\ln^3 u}{1-u}du-\int_0^1\frac{\ln^3\left( \frac{u}{2}\right)}{2-u}du\\
    &=-\frac{7\pi^4}{120}-\frac{\ln^4 2}{2}+\underbrace{\int_0^1\frac{\ln^3 u}{1-u}du}_{=-\frac{\pi^4}{15}}-\underbrace{\int_0^{\frac{1}{2}}\frac{\ln^3 u}{1-u}du}_{z=2u}-\int_0^1\frac{\ln^3\left( \frac{u}{2}\right)}{2-u}du\\
    &=-\frac{\pi^4}{8}-\frac{\ln^4 2}{2}-2\int_0^1\frac{\ln^3\left( \frac{u}{2}\right)}{2-u}du\\
    &=-\frac{\pi^4}{8}-\frac{\ln^4 2}{2}-\underbrace{\int_0^1 \frac{\ln^3 u}{1-\frac{u}{2}}du}_{-12\text{Li}_4\left(\frac{1}{2}\right)}+3\ln 2\underbrace{\int_0^1 \frac{\ln^2 u}{1-\frac{u}{2}}du}_{=\frac{21}{6}\zeta(3)-\frac{1}{3}\pi^2\ln 2+\frac{2}{3}\ln^3 2}-3\ln^2 2\underbrace{\int_0^1 \frac{\ln u}{1-\frac{u}{2}}du}_{=\ln^2 2-\frac{\pi^2}{6}}+\\&\ln^3 2\underbrace{\int_0^1 \frac{1}{1-\frac{u}{2}}du}_{=2\ln 2}\\
    &=12\text{Li}_4\left(\frac{1}{2}\right)-\frac{\pi^4}{8}-\frac{1}{2}\pi^2\ln^2 2+\frac{1}{2}\ln^4 2+\frac{21}{2}\zeta(3)\ln 2\\
    &\boxed{B=4\text{Li}_4\left(\frac{1}{2}\right)-\frac{\pi^4}{24}-\frac{1}{6}\pi^2\ln^2 2+\frac{1}{6}\ln^4 2+\frac{7}{2}\zeta(3)\ln 2}
    \end{align}
  • Fin de partie
    Modifié (January 2023)
    @Gebrane : Plus tu augmentes la puissance portant sur le logarithme plus il y aura de termes dans le développement du binôme et certains de ces termes ne seront pas nécessairement plus faciles à calculer.  Avec la méthode ci-dessus il va falloir traiter $\displaystyle \int_0^1 \frac{\ln^2 x\ln^2(1+x)}{1+x}dx$ si on veut calculer $\displaystyle \int_0^1 \frac{\ln^3 x\ln(1+x)}{1+x}dx$.
    Or, \begin{align}\int_0^1 \frac{\ln^2 x\ln^2(1+x)}{1+x}dx&\overset{\text{IPP}}=\frac{1}{3}\Big[\ln^3(1+x)\ln^2 x\Big]_0^1-\frac{2}{3}\int_0^1 \frac{\ln^3 (1+x)\ln x}{x}dx=-\frac{2}{3}\int_0^1 \frac{\ln^3 (1+x)\ln x}{x}dx\end{align}
    Bof, a priori.
    NB. Les autres termes du développement ne posent pas de problème si je vois bien.
  • Fin de partie
    Modifié (January 2023)
    La méthode utilisée pour calculer l'intégrale $J$ du premier message appliquée à $\displaystyle \int_0^1 \frac{\ln^2 x\ln^2(1+x)}{1+x}dx$ revient* à calculer \begin{align}\int_0^1\int_0^1\frac{1}{1-t}\left(\frac{\ln^2(tx)\ln(1+x)}{1+tx}-\frac{\ln^2 x\ln(1+x)}{1+x}\right)dtdx\end{align}
    La borne $1$ pour l'intégration en fonction de $x$ est gênante.
    On a envie de faire le changement de variable $u(x)=tx$ après avoir remplacé $\displaystyle \int_0^1 \frac{\ln^2 x\ln(1+x)}{1+x}dx$ par sa valeur.
    *: Il y a d'autres intégrales à calculer mais il me semble qu'elles ne posent pas de problème.
    PS.
    Plus précisément, c'est l'intégrale $\displaystyle \int_0^t \frac{\ln^2 u \ln(t+u)}{1+u}du$ qu'il va falloir calculer et qui pose problème.
    À moins que la formule de Leibniz sur la dérivée d'une intégrale à paramètre permette d'aplanir les choses. À creuser.
  • Fin de partie
    Modifié (January 2023)
    Plus précisément on a:
    \begin{align}\int_0^1 \frac{\ln^2(1+x)\ln^2 x}{1+x}dx&\overset{\text{IPP}}=\left[\left(\int_0^x \frac{\ln^2 t}{1+t}dt\right)\ln^2(1+x)\right]_0^1-2\int_0^1\frac{\ln(1+x)}{1+x}\left(\int_0^x \frac{\ln^2 t}{1+t}dt\right)dx\\
    &=\frac{3\zeta(3)\ln^2 2}{2}-2\int_0^1\int_0^1\frac{x\ln(1+x)\ln^2(tx)}{(1+tx)(1+x)}dtdx\\
    &=\frac{3\zeta(3)\ln^2 2}{2}-2\int_0^1\int_0^1\left(\frac{\ln(1+x)\ln^2(tx)}{(1-t)(1+tx)}-\frac{\ln(1+x)\ln^2(tx)}{(1-t)(1+x)}\right)dtdx\\
    &=\frac{3\zeta(3)\ln^2 2}{2}+4\Big(\int_0^1\frac{\ln t}{1-t}dt\Big)\bigg(\underbrace{\int_0^1\frac{\ln(1+x)\ln x}{1+x}dx}_{=-\frac{\zeta(3)}{8}}\bigg)\\&+2\bigg(\underbrace{\int_0^1 \frac{\ln(1+x)}{1+x}dx}_{\frac{\ln^2 2}{2}}\bigg)\bigg(\underbrace{\int_0^1 \frac{\ln^2 t}{1-t}dt}_{2\zeta(3)}\bigg)-\\&2\int_0^1\int_0^1\bigg(\frac{\ln(1+x)\ln^2(tx)}{(1-t)(1+tx)}-\frac{\ln(1+x)\ln^2 x}{(1-t)(1+x)}\bigg)dtdx.
    \end{align}
  • fjaclot
    Modifié (January 2023)
    À Fin de Partie,
    J’ai réussi à calculer l’intégrale K
    Dans le calcul de J il me manque simplement la première étape de ton calcul par IPP, ayant réussi à calculer les différentes intégrales subséquentes.
    Merci par avance et bravo pour ta persévérance.
    fjaclot;
  • gebrane
    Modifié (January 2023)
    Pour l’intégrale initiale Chatgtp me propose ceci
    \begin{align*}
    \int_0^\infty \frac{\arctan^2 x\ln^2 x}{1+x^2}dx &= \int_0^{\frac{\pi}{2}} \frac{\arctan^2(\tan(t))\ln^2(\tan(t))}{1+\tan^2(t)}\sec^2(t)dt \
    &=\int_0^{\frac{\pi}{2}} \frac{t^2\ln^2(\tan(t))}{\cos^2(t)}\sec^2(t)dt \
    &=\int_0^{\frac{\pi}{2}} t^2\ln^2(\tan(t))dt \
    \end{align*}
    Je crois que cette version gratuite est bridée, mais continuons sur son idée, donc
    $\int_0^{\frac{\pi}{2}} x^2\ln^2(\tan(x))dx=\int_0^{\pi/2} x²\ln^2(\sin x)\textrm{d}x+\int_0^{\pi/2} x^2\ln^2(\cos x)\textrm{d}x-2\int_0^{\pi/2} x^2\ln(\sin x)\ln(\cos x)\textrm{d}x$
    Est-ce que quelqu’un sait comment prouver que $$\int_{0}^{\fracπ2}x^2\log^2(\cos x)dx=\frac π {1440}[11π^4+60π^2\log^2 2+720\zeta(3)\log2$$
    Le 😄 Farceur


  • gebrane
    Modifié (January 2023)
    Merci @etanche le roi des liens.  Mais je trouve cette preuve très compliquée. La  fonction Psi est utilisée. 
    Le 😄 Farceur


  • @Etanche: C'est ce message qui m'a donné l'envie de calculer $\displaystyle \int_0^\infty \frac{\ln^2 x\arctan^2 x}{1+x^2}dx$
  • Fin de partie
    Modifié (January 2023)
    Un pas de plus, sans doute le plus difficile, \begin{align*} J&=\int_0^\infty \frac{\arctan^2 x\ln^2 x}{1+x^2}dx\\ &\overset{\text{IPP}}=\left[\left(\int_0^x \frac{\ln^2 t}{1+t^2}dt\right)\tan^2 x\right]_0^\infty-2\int_0^1 \left(\int_0^\infty\frac{x\ln^2(tx)\arctan x}{(1+x^2)(1+t^2x^2)}dx\right)dt\\ &=\frac{\pi^5}{32}-\int_0^1\left(\frac{2}{1-t^2}\underbrace{\int_0^\infty \frac{\ln^2(tx)\arctan x}{x(1+t^2x^2)}dx}_{=K(t)}-\frac{2}{1-t^2}\underbrace{\int_0^\infty \frac{\ln^2(tx)\arctan x}{x(1+x^2)}dx}_{=L(t)} \right)dt\\ K(t)&\overset{u(x)=tx}=\int_0^\infty \frac{\ln^2 u\arctan\left(\frac{u}{t}\right)}{(1+u^2)u}du\\ K^\prime(t)&=-\int_0^\infty \frac{\ln^2 u}{(1+u^2)(t^2+u^2)}du=\frac{1}{1-t^2}\int_0^\infty \frac{\ln^2 u}{1+u^2}du-\frac{1}{1-t^2}\underbrace{\int_0^\infty \frac{\ln^2 u}{t^2+u^2}du}_{z(u)=\frac{u}{t}}\\ &=\frac{\pi^3}{8(1-t^2)}-\frac{1}{t(1-t^2)}\int_0^\infty \frac{\ln^2(tz)}{1+z^2}dz\\ &=\frac{\pi^3}{8(1-t^2)}-\frac{1}{t(1-t^2)}\left(\int_0^\infty \frac{\ln^2 z}{1+z^2}dz+\int_0^\infty \frac{\ln^2 t}{1+z^2}dz+2\underbrace{\int_0^\infty \frac{\ln t\ln z}{1+z^2}dz}_{=0}\right)\\ &=-\frac{\pi^3}{8t(1+t)}-\frac{\pi\ln^2 t}{2t(1-t^2)} \end{align*} Puisque, \begin{align*} \lim_{t\rightarrow \infty}K(t)=0 \end{align*} alors, \begin{align*} K(t)&=-\left(K(\infty)-K(t)\right)=-\int_t^\infty K^{\prime}(s)ds=\int_t^\infty\left(\frac{\pi^3}{8s(1+s)}+\frac{\pi\ln^2 s}{2s(1-s^2)}\right)ds\\ &=\frac{\pi}{2}\int_t^\infty\frac{\ln^2 s}{s(1-s^2)}ds-\frac{\pi^3}{8}\ln\left(\frac{t}{1+t}\right)\\ &=\frac{\pi}{2}\int_t^1\frac{\ln^2 s}{s(1-s^2)}ds+\frac{\pi}{2}\underbrace{\int_1^\infty\frac{\ln^2 s}{s(1-s^2)}ds}_{w=\frac{1}{s^2}}-\frac{\pi^3}{8}\ln\left(\frac{t}{1+t}\right)\\ &=\frac{\pi}{2}\int_t^1\frac{\ln^2 s}{s(1-s^2)}ds-\frac{\pi}{16}\int_0^1 \frac{\ln^2 w}{1-w}dw-\frac{\pi^3}{8}\ln\left(\frac{t}{1+t}\right)\\ &=\frac{\pi}{2}\int_t^1\frac{\ln^2 s}{s(1-s^2)}ds-\frac{\pi\zeta(3)}{8}-\frac{\pi^3}{8}\ln\left(\frac{t}{1+t}\right)\\ \end{align*} \begin{align*} L(t)&=\ln^2 t\underbrace{\int_0^\infty \frac{\arctan x}{x(1+x^2)}dx}_{=A}+2\ln t\underbrace{\int_0^\infty \frac{\arctan x\ln x}{x(1+x^2)}dx}_{=B}+\underbrace{\int_0^\infty \frac{\arctan x\ln^2 x}{x(1+x^2)}dx}_{=C}\\ A&\overset{x=\tan u}=\int_0^{\frac{\pi}{2}} \frac{u}{\tan u}du\overset{\text{IPP}}=\underbrace{\Big[u\ln(\sin u)\Big]_0^{\frac{\pi}{2}}}_{=0}-\int_0^{\frac{\pi}{2}}\ln(\sin u)du=\frac{\pi}{2}\ln 2\\ B(a)&=\int_0^\infty \frac{\arctan(ax)\ln x}{x(1+x^2)}dx\\ B^\prime(a)&=\int_0^\infty \frac{\ln x}{(1+x^2)(1+a^2x^2)}dx=\frac{1}{1-a^2}\underbrace{\int_0^\infty \frac{\ln x}{1+x^2}dx}_{=0}-\frac{a}{1-a^2}\underbrace{\int_0^\infty \frac{a\ln x}{1+a^2x^2}dx}_{u(x)=ax}\\ &=-\frac{a}{1-a^2}\int_0^\infty \frac{\ln u-\ln a}{1+u^2}du=\frac{\pi a\ln a}{2(1-a^2)}\\ \end{align*} Puisque, \begin{align*} B(0)=0,B(1)=B \end{align*} alors, \begin{align*} B=\frac{\pi}{2}\int_0^1 \frac{s\ln s}{1-s^2}ds\overset{w=s^2}=\frac{\pi}{8}\int_0^1 \frac{\ln w}{1-w}dw=-\frac{\pi^3}{48} \end{align*} \begin{align*} C(a)&=\int_0^\infty \frac{\arctan(ax)\ln^2 x}{x(1+x^2)}dx\\ C^\prime(a)&=\int_0^\infty \frac{\ln^2 x}{(1+x^2)(1+a^2x^2)}dx\\ &=\frac{1}{1-a^2}\int_0^\infty \frac{\ln^2 x}{1+x^2}dx-\frac{a}{1-a^2}\underbrace{\int_0^\infty \frac{a\ln^2 x}{1+a^2x^2}dx}_{u(x)=ax}\\ &=\frac{\pi^3}{8(1-a^2)}-\frac{a}{1-a^2}\int_0^\infty \frac{\ln^2 u}{1+u^2}du+\frac{2a\ln a}{1-a^2}\int_0^\infty \frac{\ln u}{1+u^2}du-\frac{a\ln^2 a}{1-a^2}\int_0^\infty \frac{1}{1+u^2}du\\ &=\frac{\pi^3}{8(1+a)}-\frac{\pi a\ln^2 a}{2(1-a^2)} \end{align*} Puisque, \begin{align*} C(0)=0,C(1)=C \end{align*} alors, \begin{align*} C&=\int_0^1 \left(\frac{\pi^3}{8(1+s)}-\frac{\pi s\ln^2 s}{2(1-s^2)}\right)ds=\frac{\pi^3\ln 2}{8}-\frac{\pi}{2}\underbrace{\int_0^1\frac{s\ln^2 s}{1-s^2}ds}_{w=s^2}=\frac{\pi^3\ln 2}{8}-\frac{\pi}{16}\int_0^1 \frac{\ln^2 w}{1-w}dw\\&=\frac{\pi^3\ln 2}{8}-\frac{\pi\zeta(3)}{8} \end{align*} Donc, \begin{align*} L(t)&=A\ln^2 t+B\ln t+C=\frac{\pi\ln 2}{2}\ln^2 t-\frac{\pi^3}{24}\ln t+\frac{\pi^3\ln 2}{8}-\frac{\pi\zeta(3)}{8}\\ J&=\frac{\pi^5}{32}-2\int_0^1 \left(\frac{K(t)-\frac{\pi^3\ln 2}{8}+\frac{\pi\zeta(3)}{8}}{1-t^2} \right)dt+\pi\ln 2\underbrace{\int_0^1 \frac{\ln^2 t}{1-t^2}dt}_{=\frac{7\zeta(3)}{4}}-\frac{\pi^3}{12}\underbrace{\int_0^1 \frac{\ln t}{1-t^2}dt}_{=-\frac{\pi^2}{8}}\\ &=\frac{\pi^5}{24}-2\int_0^1 \left(\frac{K(t)-\frac{\pi^3\ln 2}{8}+\frac{\pi\zeta(3)}{8}}{1-t^2} \right)dt+\frac{7\pi\zeta(3)\ln 2}{4}\\ &=\frac{\pi^5}{24}+\frac{\pi^3}{4}\underbrace{\int_0^1 \frac{\ln t}{1-t^2}dt}_{=-\frac{\pi^2}{8}}-\underbrace{\pi\int_0^1 \frac{1}{1-t^2}\left(\int_t^1\frac{\ln^2 s}{s(1-s^2)}ds+\frac{\pi^2\ln (1+t)}{4}-\frac{\pi^2\ln 2}{4}\right)dt}_{\text{IPP}}+\\&\frac{7\pi\zeta(3)\ln 2}{4}\\ &=\frac{\pi^5}{96}+\frac{7\pi\zeta(3)\ln 2}{4}+\frac{\pi}{2}\int_0^1 \frac{\ln^2 t\ln\left(\frac{1-t}{1+t}\right)}{t(1-t^2)}dt-\frac{\pi^3}{8}\underbrace{\int_0^1 \frac{\ln\left(\frac{1-t}{1+t}\right)}{1+t}dt}_{u=\frac{1-t}{1+t}}\\ &=\frac{\pi^5}{96}+\frac{7\pi\zeta(3)\ln 2}{4}+\frac{\pi}{2}\int_0^1 \frac{\ln^2 t\ln\left(\frac{1-t}{1+t}\right)}{t(1-t^2)}dt-\frac{\pi^3}{8}\underbrace{\int_0^1 \frac{\ln u}{1+u}du}_{=-\frac{\pi^2}{12}}\\ &=\frac{\pi^5}{48}+\frac{7\pi\zeta(3)\ln 2}{4}+\frac{\pi}{2}\underbrace{\int_0^1 \frac{\ln^2 t\ln\left(\frac{1-t}{1+t}\right)}{t(1-t^2)}dt}_{=D} \end{align*} \begin{align*} D&=\int_0^1 \frac{t\ln^2 t\ln\left(\frac{1-t}{1+t}\right)}{1-t^2}dt+\underbrace{\int_0^1 \frac{\ln^2 t\ln\left(\frac{1-t}{1+t}\right)}{t}dt}_{\text{IPP}}\\ &=\int_0^1 \frac{t\ln^2 t\ln\left(\frac{1-t}{1+t}\right)}{1-t^2}dt+\frac{2}{3}\underbrace{\int_0^1 \frac{\ln^3 t}{1-t^2}dt}_{=-\frac{\pi^4}{16}}=\int_0^1 \frac{t\ln^2 t\ln\left(\frac{1-t}{1+t}\right)}{1-t^2}dt-\frac{\pi^4}{24}\\ D&\overset{\text{IPP}}=\underbrace{\left[\left(\int_0^t\frac{s\ln^2 s}{1-s^2}ds-\int_0^1\frac{s\ln^2 s}{(1-s^2)}ds\right)\ln\left(\frac{1-t}{1+t}\right)\right]_0^1}_{=0}+\\&2\int_0^1 \frac{1}{1-t^2}\left(\underbrace{\int_0^t\frac{s\ln^2 s}{1-s^2}ds}_{z(s)=\frac{s}{t}}-\int_0^1\frac{s\ln^2 s}{1-s^2}ds\right)dt-\frac{\pi^4}{24}\\ &=2\int_0^1 \frac{1}{1-t^2}\left(\int_0^1 \frac{t^2z\ln^2(tz)}{1-t^2z^2}dz-\int_0^1\frac{s\ln^2 s}{1-s^2}ds\right)dt-\frac{\pi^4}{24}\\ &=2\int_0^1\int_0^1\left(\frac{s\ln^2(st)}{(1-s^2)(1-t^2)}-\frac{s\ln^2(st)}{(1-s^2)(1-s^2t^2)}-\frac{s\ln^2 s}{(1-t^2)(1-s^2)}\right)dsdt-\frac{\pi^4}{24}\\ &=4\left(\underbrace{\int_0^1 \frac{s\ln s }{1-s^2}ds}_{=-\frac{\pi^2}{24}}\right)\left(\underbrace{\int_0^1 \frac{\ln t}{1-t^2}dt}_{=-\frac{\pi^2}{8}}\right)+2\int_0^1\int_0^1\left(\frac{s\ln^2 t}{(1-s^2)(1-t^2)}-\frac{s\ln^2(st)}{(1-s^2)(1-s^2t^2)}\right)dsdt-\\&\frac{\pi^4}{24}\\ &=2\underbrace{\int_0^1\frac{1}{1-s^2}\left(\int_s^1 \frac{\ln^2 t}{1-t^2}dt\right)ds}_{\text{IPP}}+2\int_0^1\int_0^1\left(\frac{s\ln^2 t}{(1-s^2)(1-t^2)}-\frac{\ln^2 t}{(1-s^2)(1-t^2)}\right)-\frac{\pi^4}{48}\\ &=-\int_0^1 \frac{\ln\left(\frac{1-s}{1+s}\right)\ln^2 s}{1-s^2}ds-2\left(\int_0^1 \frac{1}{1+s}ds\right)\left(\underbrace{\int_0^1 \frac{\ln^2 t}{1-t^2}dt}_{=\frac{7\zeta(3)}{4}}\right)-\frac{\pi^4}{48}\\ &=\boxed{-\int_0^1 \frac{\ln\left(\frac{1-s}{1+s}\right)\ln^2 s}{1-s^2}ds-\frac{7\zeta(3)\ln 2}{2}-\frac{\pi^4}{48}} \end{align*} Donc, \begin{align*}\boxed{J=\frac{\pi^5}{96}-\frac{\pi}{2}\int_0^1 \frac{\ln\left(\frac{1-s}{1+s}\right)\ln^2 s}{1-s^2}ds}\end{align*}
  • C'est du solide
    Le 😄 Farceur


  • fjaclot
    Modifié (January 2023)
    À Fin de Partie.
    Merci. Impressionnant !
    Il faut vraiment de la persévérance pour poursuivre ce calcul sans être sûr a priori qu’il aboutisse à un résultat intermédiaire assez simple.
    Au total même si j’admire ton agilité et ta persévérance je me demande si l’approche suggérée par gebrane et etanche n’est pas plus rapide.
    fjaclot;
  • Fin de partie
    Modifié (January 2023)
    @Fjaclot. Tu fais comment pour calculer :
    \begin{align}\int_0^{\pi/2} x^2\ln(\sin x)\ln(\cos x)\textrm{d}x\quad?\end{align}
  • gebrane
    Modifié (January 2023)
    FDP  C'est simple, elle s'exprime à l'aide des deux autres et d'autres calculables
    utilise que $ab=\dfrac{(a+b)^2-a^2-b^2}2$
    Le 😄 Farceur


  • Fin de partie
    Modifié (January 2023)
    @Gebrane:  J'aurais dû mieux poser ma question.
    Si tu ne sais pas calculer $\displaystyle \int_0^\infty \frac{\ln^2 x\arctan^2 x}{1+x^2}dx$ ?
    Il est vrai qu'on a aussi :
    \begin{align}\int_0^{\frac{\pi}{2}}x^2\ln^2\left(2\sin x\cos x\right)dx
    &=\int_0^{\frac{\pi}{2}}x^2\ln^2\left(\sin(2x)\right)dx\\
    &=\frac{1}{8}\int_0^\pi u^2\ln^2\left(\sin u\right)du\\
    &=\frac{1}{8}\int_0^{\frac{\pi}{2}} u^2\ln^2\left(\sin u\right)du+\frac{1}{8}\underbrace{\int_{\frac{\pi}{2}}^\pi u^2\ln^2\left(\sin u\right)du}_{z=\pi-u}\\
    &=\frac{1}{8}\int_0^{\frac{\pi}{2}} u^2\ln^2\left(\sin u\right)du+\frac{1}{8}\int_0^{\frac{\pi}{2}} \left(\pi-z\right)^2\ln^2\left(\sin z\right)dz\\
    &=\frac{1}{4}\int_0^{\frac{\pi}{2}} u^2\ln^2\left(\sin u\right)du+\frac{\pi^2}{8}\int_0^{\frac{\pi}{2}} \ln^2\left(\sin z\right)dz-\frac{\pi}{4}\int_0^{\frac{\pi}{2}} z\ln^2\left(\sin z\right)dz
    \end{align} et,\begin{align}\int_0^{\frac{\pi}{2}}x^2\ln^2\left(2\sin x\cos x\right)dx&=\int_0^{\frac{\pi}{2}}x^2\ln^2\left(\sin x\cos x\right)dx+\ln^2 2\int_0^{\frac{\pi}{2}}x^2dx+\\&
    2\ln 2\int_0^{\frac{\pi}{2}}x^2\ln\left(\frac{\sin(2x)}{2}\right)dx\\
    &=\int_0^{\frac{\pi}{2}}x^2\ln^2\left(\sin x\cos x\right)dx+\frac{1}{4}\ln 2\int_0^{\pi}x^2\ln\left(\sin x\right)dx-\frac{\pi^3\ln^2 2}{24}
    \end{align}
  • @Fin de partie  Bonjour;  Je ne comprends ce que tu essayais  de me dire dans ton dernier message. Mon dernier message disait que
    $\int_0^{\frac{\pi}{2}} x^2\ln^2(\tan(x))dx=\int_0^{\pi/2} x²\ln^2(\sin x)\textrm{d}x+\int_0^{\pi/2} x^2\ln^2(\cos x)\textrm{d}x-2\int_0^{\pi/2} x^2\ln(\sin x)\ln(\cos x)\textrm{d}x=2\int_0^{\pi/2} x²\ln^2(\sin x)\textrm{d}x+2\int_0^{\pi/2} x^2\ln^2(\cos x)\textrm{d}x-\int_0^{\pi/2} x²\ln^2(\frac 12\sin 2x)\textrm{d}x$ et
    $\int_0^{\pi/2} x²\ln^2(\frac 12\sin 2x)\textrm{d}x=\int_0^{\pi/2} x²\ln^2(\frac 12)\textrm{d}x+\int_0^{\pi/2} x²\ln^2(\sin 2x)\textrm{d}x+\int_0^{\pi/2} x²\ln(\sin 2x)\textrm{d}x$







    Le 😄 Farceur


  • Fin de partie
    Modifié (January 2023)
    A priori, la méthode développée plus haut pourrait servir pour calculer : \begin{align}\int_0^\infty \frac{\arctan^2 x\ln^4 x}{1+x^2}dx\end{align}
    (je n'ai fait aucun calcul donc il faut rester prudent sur la faisabilité)
  • C'est ce qu'on appelle le dialogue des sourds :/
    Le 😄 Farceur


  • Fin de partie
    Modifié (January 2023)
    @Gebrane: Ce fil est né de la lecture de la question posée sur MathExchange: le calcul de $\displaystyle \int_0^{\frac{\pi}{2}}x^2\ln^2(\sin x)dx$
    L'idée de départ était le truc standard, on a $\displaystyle \int_0^{\frac{\pi}{2}}x^2\ln^2(\tan x)dx+\int_0^{\frac{\pi}{2}}x^2\ln^2(\sin x\cos x)dx=2\int_0^{\frac{\pi}{2}}x^2\ln^2(\sin x)dx+2\int_0^{\frac{\pi}{2}}x^2\ln^2(\cos x)dx$
    "Modulo" quelques intégrales plus simples à calculer si on sait calculer $\displaystyle \int_0^{\frac{\pi}{2}}x^2\ln^2(\tan x)dx$ on sait calculer $\displaystyle \int_0^{\frac{\pi}{2}}x^2\ln^2(\sin x)dx$ et réciproquement.
    PS.
    Maintenant, si on considère $\displaystyle \int_0^{\frac{\pi}{2}}x^2\ln^4(\tan x)dx$ on ne va PAS pouvoir considérer une astuce aussi simple pour mener le calcul. Par contre, a priori, la méthode montrée plus haut, pourrait fonctionner encore très bien (il faut être prudent toutefois, je n'ai pas fait les calculs jusqu'au bout).
  • Ok
    Le 😄 Farceur


Connectez-vous ou Inscrivez-vous pour répondre.